Sequence with anchoring property is convergent in finite dimension












1












$begingroup$


Let $(x_k)$ be a sequence in $mathbb{R}^n$. Furthermore, assume that the sequence has the following "anchoring" property: $exists y,z inmathbb{R}^n$ such that $lim|x_k-y|=r_y$ and $lim|x_k-z|=r_z$. Show that the sequence $(x_k)$ is convergent.



I have shown it's bounded:



$|x_k| leq |x_k-y|+|y| < r_y + varepsilon + |y| < infty$.



From here I am not sure where to go. Any hints? I thought maybe I should use the sequential compactness of closed balls in $mathbb{R}^n$ to find a convergent subsequence maybe?










share|cite|improve this question









$endgroup$








  • 1




    $begingroup$
    are there other conditions such as $y ne z$
    $endgroup$
    – Siong Thye Goh
    Dec 11 '18 at 11:05












  • $begingroup$
    Yeah $yneq z$ of course otherwise it's not true (circle)
    $endgroup$
    – Tony S.F.
    Dec 11 '18 at 11:09










  • $begingroup$
    Given the answer below, it seems to be true only for $n=1$.
    $endgroup$
    – daw
    Dec 11 '18 at 12:15










  • $begingroup$
    Shouldn't be $y,zin Bbb R$?
    $endgroup$
    – Mostafa Ayaz
    Dec 11 '18 at 23:14












  • $begingroup$
    @MostafaAyaz they are in R^n. OP writes |y|, but we understand than he/she means the norm of a vector.
    $endgroup$
    – GNUSupporter 8964民主女神 地下教會
    Dec 12 '18 at 13:41


















1












$begingroup$


Let $(x_k)$ be a sequence in $mathbb{R}^n$. Furthermore, assume that the sequence has the following "anchoring" property: $exists y,z inmathbb{R}^n$ such that $lim|x_k-y|=r_y$ and $lim|x_k-z|=r_z$. Show that the sequence $(x_k)$ is convergent.



I have shown it's bounded:



$|x_k| leq |x_k-y|+|y| < r_y + varepsilon + |y| < infty$.



From here I am not sure where to go. Any hints? I thought maybe I should use the sequential compactness of closed balls in $mathbb{R}^n$ to find a convergent subsequence maybe?










share|cite|improve this question









$endgroup$








  • 1




    $begingroup$
    are there other conditions such as $y ne z$
    $endgroup$
    – Siong Thye Goh
    Dec 11 '18 at 11:05












  • $begingroup$
    Yeah $yneq z$ of course otherwise it's not true (circle)
    $endgroup$
    – Tony S.F.
    Dec 11 '18 at 11:09










  • $begingroup$
    Given the answer below, it seems to be true only for $n=1$.
    $endgroup$
    – daw
    Dec 11 '18 at 12:15










  • $begingroup$
    Shouldn't be $y,zin Bbb R$?
    $endgroup$
    – Mostafa Ayaz
    Dec 11 '18 at 23:14












  • $begingroup$
    @MostafaAyaz they are in R^n. OP writes |y|, but we understand than he/she means the norm of a vector.
    $endgroup$
    – GNUSupporter 8964民主女神 地下教會
    Dec 12 '18 at 13:41
















1












1








1





$begingroup$


Let $(x_k)$ be a sequence in $mathbb{R}^n$. Furthermore, assume that the sequence has the following "anchoring" property: $exists y,z inmathbb{R}^n$ such that $lim|x_k-y|=r_y$ and $lim|x_k-z|=r_z$. Show that the sequence $(x_k)$ is convergent.



I have shown it's bounded:



$|x_k| leq |x_k-y|+|y| < r_y + varepsilon + |y| < infty$.



From here I am not sure where to go. Any hints? I thought maybe I should use the sequential compactness of closed balls in $mathbb{R}^n$ to find a convergent subsequence maybe?










share|cite|improve this question









$endgroup$




Let $(x_k)$ be a sequence in $mathbb{R}^n$. Furthermore, assume that the sequence has the following "anchoring" property: $exists y,z inmathbb{R}^n$ such that $lim|x_k-y|=r_y$ and $lim|x_k-z|=r_z$. Show that the sequence $(x_k)$ is convergent.



I have shown it's bounded:



$|x_k| leq |x_k-y|+|y| < r_y + varepsilon + |y| < infty$.



From here I am not sure where to go. Any hints? I thought maybe I should use the sequential compactness of closed balls in $mathbb{R}^n$ to find a convergent subsequence maybe?







real-analysis sequences-and-series functional-analysis






share|cite|improve this question













share|cite|improve this question











share|cite|improve this question




share|cite|improve this question










asked Dec 11 '18 at 10:58









Tony S.F.Tony S.F.

3,29821028




3,29821028








  • 1




    $begingroup$
    are there other conditions such as $y ne z$
    $endgroup$
    – Siong Thye Goh
    Dec 11 '18 at 11:05












  • $begingroup$
    Yeah $yneq z$ of course otherwise it's not true (circle)
    $endgroup$
    – Tony S.F.
    Dec 11 '18 at 11:09










  • $begingroup$
    Given the answer below, it seems to be true only for $n=1$.
    $endgroup$
    – daw
    Dec 11 '18 at 12:15










  • $begingroup$
    Shouldn't be $y,zin Bbb R$?
    $endgroup$
    – Mostafa Ayaz
    Dec 11 '18 at 23:14












  • $begingroup$
    @MostafaAyaz they are in R^n. OP writes |y|, but we understand than he/she means the norm of a vector.
    $endgroup$
    – GNUSupporter 8964民主女神 地下教會
    Dec 12 '18 at 13:41
















  • 1




    $begingroup$
    are there other conditions such as $y ne z$
    $endgroup$
    – Siong Thye Goh
    Dec 11 '18 at 11:05












  • $begingroup$
    Yeah $yneq z$ of course otherwise it's not true (circle)
    $endgroup$
    – Tony S.F.
    Dec 11 '18 at 11:09










  • $begingroup$
    Given the answer below, it seems to be true only for $n=1$.
    $endgroup$
    – daw
    Dec 11 '18 at 12:15










  • $begingroup$
    Shouldn't be $y,zin Bbb R$?
    $endgroup$
    – Mostafa Ayaz
    Dec 11 '18 at 23:14












  • $begingroup$
    @MostafaAyaz they are in R^n. OP writes |y|, but we understand than he/she means the norm of a vector.
    $endgroup$
    – GNUSupporter 8964民主女神 地下教會
    Dec 12 '18 at 13:41










1




1




$begingroup$
are there other conditions such as $y ne z$
$endgroup$
– Siong Thye Goh
Dec 11 '18 at 11:05






$begingroup$
are there other conditions such as $y ne z$
$endgroup$
– Siong Thye Goh
Dec 11 '18 at 11:05














$begingroup$
Yeah $yneq z$ of course otherwise it's not true (circle)
$endgroup$
– Tony S.F.
Dec 11 '18 at 11:09




$begingroup$
Yeah $yneq z$ of course otherwise it's not true (circle)
$endgroup$
– Tony S.F.
Dec 11 '18 at 11:09












$begingroup$
Given the answer below, it seems to be true only for $n=1$.
$endgroup$
– daw
Dec 11 '18 at 12:15




$begingroup$
Given the answer below, it seems to be true only for $n=1$.
$endgroup$
– daw
Dec 11 '18 at 12:15












$begingroup$
Shouldn't be $y,zin Bbb R$?
$endgroup$
– Mostafa Ayaz
Dec 11 '18 at 23:14






$begingroup$
Shouldn't be $y,zin Bbb R$?
$endgroup$
– Mostafa Ayaz
Dec 11 '18 at 23:14














$begingroup$
@MostafaAyaz they are in R^n. OP writes |y|, but we understand than he/she means the norm of a vector.
$endgroup$
– GNUSupporter 8964民主女神 地下教會
Dec 12 '18 at 13:41






$begingroup$
@MostafaAyaz they are in R^n. OP writes |y|, but we understand than he/she means the norm of a vector.
$endgroup$
– GNUSupporter 8964民主女神 地下教會
Dec 12 '18 at 13:41












1 Answer
1






active

oldest

votes


















2












$begingroup$

Even though you have $y ne z$, the statement is still incorrect. For a counterexample, take $n = 2$, and two intersecting circles centred at $y$ and $z$ with radii $r_y$ and $r_z$ respectively. Take $(x_k)_k$ to be the sequence oscillating between the two points of intersection. From the uniqueness of limit in Hausdorff spaces (e.g. $Bbb{R}^n$), we know that $(x_k)_k$ can't be convergent.



$$
%
defplace#1#2#3{smash{rlap{hskip{#1em}raise{#2em}{#3}}}}
%
hskip 1em
%
place{-3}{-0.5}{Hugebigcirc}
place{-3}{0}{bullet_{large y}}
place{-1}{-0.5}{Hugebigcirc}
place{0.5}{0}{bullet_{large z}}
$$






share|cite|improve this answer











$endgroup$









  • 1




    $begingroup$
    agreed, if it converges it does in the intersection of two spheres but if they are not tangent it does not converges (but a sub sequence does)
    $endgroup$
    – Alexis
    Dec 11 '18 at 11:14










  • $begingroup$
    You're right, but what if, instead of just two points $y$ and $z$, we assume the following instead: $exists Ssubset mathbb{R}^n$ such that $forall sin S, lim|x_n-s|=r_s$ and int$Sneqemptyset$.
    $endgroup$
    – Tony S.F.
    Dec 12 '18 at 11:14












  • $begingroup$
    You may try to see if it's Cauchy. I think that's true, but I don't know how to prove that. Why not start a new question, so that others can upvote and answer your question.
    $endgroup$
    – GNUSupporter 8964民主女神 地下教會
    Dec 12 '18 at 13:56










  • $begingroup$
    I will, but comments are partially intended for this sort of informal brainstorming iirc. Its possible you could answer decisively with a short reason and then i could revise the question to a more meaningful form before posting again. This would avoid too many similar posts.
    $endgroup$
    – Tony S.F.
    Dec 12 '18 at 19:00













Your Answer





StackExchange.ifUsing("editor", function () {
return StackExchange.using("mathjaxEditing", function () {
StackExchange.MarkdownEditor.creationCallbacks.add(function (editor, postfix) {
StackExchange.mathjaxEditing.prepareWmdForMathJax(editor, postfix, [["$", "$"], ["\\(","\\)"]]);
});
});
}, "mathjax-editing");

StackExchange.ready(function() {
var channelOptions = {
tags: "".split(" "),
id: "69"
};
initTagRenderer("".split(" "), "".split(" "), channelOptions);

StackExchange.using("externalEditor", function() {
// Have to fire editor after snippets, if snippets enabled
if (StackExchange.settings.snippets.snippetsEnabled) {
StackExchange.using("snippets", function() {
createEditor();
});
}
else {
createEditor();
}
});

function createEditor() {
StackExchange.prepareEditor({
heartbeatType: 'answer',
autoActivateHeartbeat: false,
convertImagesToLinks: true,
noModals: true,
showLowRepImageUploadWarning: true,
reputationToPostImages: 10,
bindNavPrevention: true,
postfix: "",
imageUploader: {
brandingHtml: "Powered by u003ca class="icon-imgur-white" href="https://imgur.com/"u003eu003c/au003e",
contentPolicyHtml: "User contributions licensed under u003ca href="https://creativecommons.org/licenses/by-sa/3.0/"u003ecc by-sa 3.0 with attribution requiredu003c/au003e u003ca href="https://stackoverflow.com/legal/content-policy"u003e(content policy)u003c/au003e",
allowUrls: true
},
noCode: true, onDemand: true,
discardSelector: ".discard-answer"
,immediatelyShowMarkdownHelp:true
});


}
});














draft saved

draft discarded


















StackExchange.ready(
function () {
StackExchange.openid.initPostLogin('.new-post-login', 'https%3a%2f%2fmath.stackexchange.com%2fquestions%2f3035174%2fsequence-with-anchoring-property-is-convergent-in-finite-dimension%23new-answer', 'question_page');
}
);

Post as a guest















Required, but never shown

























1 Answer
1






active

oldest

votes








1 Answer
1






active

oldest

votes









active

oldest

votes






active

oldest

votes









2












$begingroup$

Even though you have $y ne z$, the statement is still incorrect. For a counterexample, take $n = 2$, and two intersecting circles centred at $y$ and $z$ with radii $r_y$ and $r_z$ respectively. Take $(x_k)_k$ to be the sequence oscillating between the two points of intersection. From the uniqueness of limit in Hausdorff spaces (e.g. $Bbb{R}^n$), we know that $(x_k)_k$ can't be convergent.



$$
%
defplace#1#2#3{smash{rlap{hskip{#1em}raise{#2em}{#3}}}}
%
hskip 1em
%
place{-3}{-0.5}{Hugebigcirc}
place{-3}{0}{bullet_{large y}}
place{-1}{-0.5}{Hugebigcirc}
place{0.5}{0}{bullet_{large z}}
$$






share|cite|improve this answer











$endgroup$









  • 1




    $begingroup$
    agreed, if it converges it does in the intersection of two spheres but if they are not tangent it does not converges (but a sub sequence does)
    $endgroup$
    – Alexis
    Dec 11 '18 at 11:14










  • $begingroup$
    You're right, but what if, instead of just two points $y$ and $z$, we assume the following instead: $exists Ssubset mathbb{R}^n$ such that $forall sin S, lim|x_n-s|=r_s$ and int$Sneqemptyset$.
    $endgroup$
    – Tony S.F.
    Dec 12 '18 at 11:14












  • $begingroup$
    You may try to see if it's Cauchy. I think that's true, but I don't know how to prove that. Why not start a new question, so that others can upvote and answer your question.
    $endgroup$
    – GNUSupporter 8964民主女神 地下教會
    Dec 12 '18 at 13:56










  • $begingroup$
    I will, but comments are partially intended for this sort of informal brainstorming iirc. Its possible you could answer decisively with a short reason and then i could revise the question to a more meaningful form before posting again. This would avoid too many similar posts.
    $endgroup$
    – Tony S.F.
    Dec 12 '18 at 19:00


















2












$begingroup$

Even though you have $y ne z$, the statement is still incorrect. For a counterexample, take $n = 2$, and two intersecting circles centred at $y$ and $z$ with radii $r_y$ and $r_z$ respectively. Take $(x_k)_k$ to be the sequence oscillating between the two points of intersection. From the uniqueness of limit in Hausdorff spaces (e.g. $Bbb{R}^n$), we know that $(x_k)_k$ can't be convergent.



$$
%
defplace#1#2#3{smash{rlap{hskip{#1em}raise{#2em}{#3}}}}
%
hskip 1em
%
place{-3}{-0.5}{Hugebigcirc}
place{-3}{0}{bullet_{large y}}
place{-1}{-0.5}{Hugebigcirc}
place{0.5}{0}{bullet_{large z}}
$$






share|cite|improve this answer











$endgroup$









  • 1




    $begingroup$
    agreed, if it converges it does in the intersection of two spheres but if they are not tangent it does not converges (but a sub sequence does)
    $endgroup$
    – Alexis
    Dec 11 '18 at 11:14










  • $begingroup$
    You're right, but what if, instead of just two points $y$ and $z$, we assume the following instead: $exists Ssubset mathbb{R}^n$ such that $forall sin S, lim|x_n-s|=r_s$ and int$Sneqemptyset$.
    $endgroup$
    – Tony S.F.
    Dec 12 '18 at 11:14












  • $begingroup$
    You may try to see if it's Cauchy. I think that's true, but I don't know how to prove that. Why not start a new question, so that others can upvote and answer your question.
    $endgroup$
    – GNUSupporter 8964民主女神 地下教會
    Dec 12 '18 at 13:56










  • $begingroup$
    I will, but comments are partially intended for this sort of informal brainstorming iirc. Its possible you could answer decisively with a short reason and then i could revise the question to a more meaningful form before posting again. This would avoid too many similar posts.
    $endgroup$
    – Tony S.F.
    Dec 12 '18 at 19:00
















2












2








2





$begingroup$

Even though you have $y ne z$, the statement is still incorrect. For a counterexample, take $n = 2$, and two intersecting circles centred at $y$ and $z$ with radii $r_y$ and $r_z$ respectively. Take $(x_k)_k$ to be the sequence oscillating between the two points of intersection. From the uniqueness of limit in Hausdorff spaces (e.g. $Bbb{R}^n$), we know that $(x_k)_k$ can't be convergent.



$$
%
defplace#1#2#3{smash{rlap{hskip{#1em}raise{#2em}{#3}}}}
%
hskip 1em
%
place{-3}{-0.5}{Hugebigcirc}
place{-3}{0}{bullet_{large y}}
place{-1}{-0.5}{Hugebigcirc}
place{0.5}{0}{bullet_{large z}}
$$






share|cite|improve this answer











$endgroup$



Even though you have $y ne z$, the statement is still incorrect. For a counterexample, take $n = 2$, and two intersecting circles centred at $y$ and $z$ with radii $r_y$ and $r_z$ respectively. Take $(x_k)_k$ to be the sequence oscillating between the two points of intersection. From the uniqueness of limit in Hausdorff spaces (e.g. $Bbb{R}^n$), we know that $(x_k)_k$ can't be convergent.



$$
%
defplace#1#2#3{smash{rlap{hskip{#1em}raise{#2em}{#3}}}}
%
hskip 1em
%
place{-3}{-0.5}{Hugebigcirc}
place{-3}{0}{bullet_{large y}}
place{-1}{-0.5}{Hugebigcirc}
place{0.5}{0}{bullet_{large z}}
$$







share|cite|improve this answer














share|cite|improve this answer



share|cite|improve this answer








edited Dec 11 '18 at 11:21

























answered Dec 11 '18 at 11:10









GNUSupporter 8964民主女神 地下教會GNUSupporter 8964民主女神 地下教會

13.3k72549




13.3k72549








  • 1




    $begingroup$
    agreed, if it converges it does in the intersection of two spheres but if they are not tangent it does not converges (but a sub sequence does)
    $endgroup$
    – Alexis
    Dec 11 '18 at 11:14










  • $begingroup$
    You're right, but what if, instead of just two points $y$ and $z$, we assume the following instead: $exists Ssubset mathbb{R}^n$ such that $forall sin S, lim|x_n-s|=r_s$ and int$Sneqemptyset$.
    $endgroup$
    – Tony S.F.
    Dec 12 '18 at 11:14












  • $begingroup$
    You may try to see if it's Cauchy. I think that's true, but I don't know how to prove that. Why not start a new question, so that others can upvote and answer your question.
    $endgroup$
    – GNUSupporter 8964民主女神 地下教會
    Dec 12 '18 at 13:56










  • $begingroup$
    I will, but comments are partially intended for this sort of informal brainstorming iirc. Its possible you could answer decisively with a short reason and then i could revise the question to a more meaningful form before posting again. This would avoid too many similar posts.
    $endgroup$
    – Tony S.F.
    Dec 12 '18 at 19:00
















  • 1




    $begingroup$
    agreed, if it converges it does in the intersection of two spheres but if they are not tangent it does not converges (but a sub sequence does)
    $endgroup$
    – Alexis
    Dec 11 '18 at 11:14










  • $begingroup$
    You're right, but what if, instead of just two points $y$ and $z$, we assume the following instead: $exists Ssubset mathbb{R}^n$ such that $forall sin S, lim|x_n-s|=r_s$ and int$Sneqemptyset$.
    $endgroup$
    – Tony S.F.
    Dec 12 '18 at 11:14












  • $begingroup$
    You may try to see if it's Cauchy. I think that's true, but I don't know how to prove that. Why not start a new question, so that others can upvote and answer your question.
    $endgroup$
    – GNUSupporter 8964民主女神 地下教會
    Dec 12 '18 at 13:56










  • $begingroup$
    I will, but comments are partially intended for this sort of informal brainstorming iirc. Its possible you could answer decisively with a short reason and then i could revise the question to a more meaningful form before posting again. This would avoid too many similar posts.
    $endgroup$
    – Tony S.F.
    Dec 12 '18 at 19:00










1




1




$begingroup$
agreed, if it converges it does in the intersection of two spheres but if they are not tangent it does not converges (but a sub sequence does)
$endgroup$
– Alexis
Dec 11 '18 at 11:14




$begingroup$
agreed, if it converges it does in the intersection of two spheres but if they are not tangent it does not converges (but a sub sequence does)
$endgroup$
– Alexis
Dec 11 '18 at 11:14












$begingroup$
You're right, but what if, instead of just two points $y$ and $z$, we assume the following instead: $exists Ssubset mathbb{R}^n$ such that $forall sin S, lim|x_n-s|=r_s$ and int$Sneqemptyset$.
$endgroup$
– Tony S.F.
Dec 12 '18 at 11:14






$begingroup$
You're right, but what if, instead of just two points $y$ and $z$, we assume the following instead: $exists Ssubset mathbb{R}^n$ such that $forall sin S, lim|x_n-s|=r_s$ and int$Sneqemptyset$.
$endgroup$
– Tony S.F.
Dec 12 '18 at 11:14














$begingroup$
You may try to see if it's Cauchy. I think that's true, but I don't know how to prove that. Why not start a new question, so that others can upvote and answer your question.
$endgroup$
– GNUSupporter 8964民主女神 地下教會
Dec 12 '18 at 13:56




$begingroup$
You may try to see if it's Cauchy. I think that's true, but I don't know how to prove that. Why not start a new question, so that others can upvote and answer your question.
$endgroup$
– GNUSupporter 8964民主女神 地下教會
Dec 12 '18 at 13:56












$begingroup$
I will, but comments are partially intended for this sort of informal brainstorming iirc. Its possible you could answer decisively with a short reason and then i could revise the question to a more meaningful form before posting again. This would avoid too many similar posts.
$endgroup$
– Tony S.F.
Dec 12 '18 at 19:00






$begingroup$
I will, but comments are partially intended for this sort of informal brainstorming iirc. Its possible you could answer decisively with a short reason and then i could revise the question to a more meaningful form before posting again. This would avoid too many similar posts.
$endgroup$
– Tony S.F.
Dec 12 '18 at 19:00




















draft saved

draft discarded




















































Thanks for contributing an answer to Mathematics Stack Exchange!


  • Please be sure to answer the question. Provide details and share your research!

But avoid



  • Asking for help, clarification, or responding to other answers.

  • Making statements based on opinion; back them up with references or personal experience.


Use MathJax to format equations. MathJax reference.


To learn more, see our tips on writing great answers.




draft saved


draft discarded














StackExchange.ready(
function () {
StackExchange.openid.initPostLogin('.new-post-login', 'https%3a%2f%2fmath.stackexchange.com%2fquestions%2f3035174%2fsequence-with-anchoring-property-is-convergent-in-finite-dimension%23new-answer', 'question_page');
}
);

Post as a guest















Required, but never shown





















































Required, but never shown














Required, but never shown












Required, but never shown







Required, but never shown

































Required, but never shown














Required, but never shown












Required, but never shown







Required, but never shown







Popular posts from this blog

Plaza Victoria

In PowerPoint, is there a keyboard shortcut for bulleted / numbered list?

How to put 3 figures in Latex with 2 figures side by side and 1 below these side by side images but in...